What if you draw a line from the resting position of the wave to the crest of the wave, what would your line indicate?


A.
It indicates the amplitude of the wave which decreases when higher energy is put in the wave.


B.
It indicates the wavelength of the wave which increases when higher energy is put in the wave.


C.
It indicates the amplitude of the wave which increases when higher energy is put in the wave.

Answers

Answer 1

Answer:

The correct answer is C. It indicates the amplitude of the wave which increases when higher energy is put in the wave.

Explanation:

The amplitude of a wave is the maximum displacement of a particle from its resting position as the wave passes through it. Drawing a line from the resting position to the crest of the wave indicates the maximum displacement of the particle from its resting position, which is the amplitude of the wave.When higher energy is put into a wave, the amplitude of the wave increases. This is because the energy of the wave is directly proportional to its amplitude. Therefore, if more energy is put into the wave, the amplitude increases.


Related Questions

in simple harmonic motion, when is the magnitude of the acceleration the greatest? (there could be more than one correct choice.)

Answers

In simple harmonic motion, the magnitude of the acceleration is maximum when the displacement is maximum, which is at the equilibrium position.

Simple harmonic motion (SHM) is a form of motion in which an object oscillates (moves back and forth) under a restoring force that is proportional to the object's displacement from its equilibrium position. The object moves towards its equilibrium position under the influence of this force when it is displaced from its equilibrium position. In a spring-mass system, for example, when a spring is stretched or compressed, a restoring force proportional to the amount of stretching or compression is created. When the spring is released, the restoring force pushes the mass back toward its equilibrium position, causing it to oscillate back and forth. There are numerous examples of SHM in daily life, including the motion of a simple pendulum and the motion of a mass attached to a spring. The magnitude of the acceleration is maximum when the displacement is maximum, i.e., at the equilibrium position.

To learn more about Simple harmonic motion :

https://brainly.com/question/27237546

#SPJ11

3. Large amplitude vibrations produced when the of receiver of the applied forced vibration matches the

Answers

An object's amplitude dramatically increases when the frequency of the applied forced vibrations matches the object's natural frequency. Resonance describes this behavior.

Theory A wave's amplitude directly relates to the quantity of energy it can carry. A wave with a high amplitude carries a lot of energy, whereas one with a low amplitude carries only a little. A wave's strength is determined by the typical energy that moves through a given area in a certain amount of time and in a particular direction.The sound wave's amplitude grows in proportion to its strength. We perceive louder noises to be of higher intensity. Comparative sound intensities are frequently expressed using decibels (dB)

For more information on amplitude of vibration kindly visit to

https://brainly.com/question/1380029

#SPJ1

Black hole A has a mass that is twice the mass of black hole B. From this information, you can say that the event horizon of black hole A isa. larger than the event horizon of black hole B.b. smaller than the event horizon of black hole B.c. the same size as the event horizon of black hole B.

Answers

Since it is the point of no return when the gravitational pull is so intense that not even light can escape, the event horizon of a black hole is directly correlated to its mass.

Consequently, a broader event horizon would be present around a black hole with a higher mass than one with a lower mass.

Steps

We can infer that black hole A's event horizon is greater than black hole B's event horizon since black hole A is twice as massive as black hole B.

This is because black hole A's bigger mass makes its gravitational pull stronger, and because this stronger gravitational attraction spreads farther from the black hole, it creates a larger event.

The event horizon is the region surrounding a black hole beyond which nothing—not even light—can exist because of the black hole's powerful gravitational pull. It is intimately correlated with the black hole's mass, with wider event horizons being associated with larger black holes.

According to the scenario, black hole A is twice as massive as black hole B. This indicates that because black hole A is more massive than black hole B, its gravitational attraction is stronger.

The black hole's event horizon is greater than black hole B's because of the stronger gravitational force that reaches further from it.

learn more about Black hole here

https://brainly.com/question/6037502

#SPJ1

A 120.00 kg roller-coaster car is pressed against a spring of constant 925 N/m and compresses it 3.00 meters. It is then released and rolls up an inclined portion of the track. How high up the incline will car roll before coming to a stop?
answer with correct units​

Answers

Answer:

Explanation:

The potential energy stored in the compressed spring is given by:

PE = (1/2) k x^2

where:

k = spring constant = 925 N/m

x = compression of the spring = 3.00 m

Substituting the values, we get:

PE = (1/2) (925 N/m) (3.00 m)^2 = 4162.5 J

At the bottom of the incline, the roller-coaster car has both potential energy (PE) and kinetic energy (KE). At the top of the incline, the roller-coaster car will have only potential energy, because it has come to a stop. We can therefore set the PE at the bottom equal to the PE at the top:

PE_bottom = PE_top

where:

PE_bottom = m g h, where m is the mass of the roller-coaster car, g is the acceleration due to gravity (9.81 m/s^2), and h is the height of the incline

PE_top = 4162.5 J, the potential energy stored in the compressed spring

Substituting the values, we get:

m g h = 4162.5 J

Solving for h, we get:

h = 4162.5 J / (m g) = 4162.5 J / (120.00 kg x 9.81 m/s^2) ≈ 3.54 m

Therefore, the roller-coaster car will roll up the incline to a height of approximately 3.54 meters before coming to a stop.

The roller-coaster car will roll up approximately 7.08 meters up the incline before coming to a stop.

To calculate how high up the incline the roller-coaster car will roll before coming to a stop, we can use the principle of conservation of mechanical energy. At the initial position, the roller-coaster car has potential energy stored in the compressed spring, and at the highest point on the incline, it will have only potential energy due to its height.

The total mechanical energy at the initial position is the sum of the potential energy stored in the compressed spring and the kinetic energy of the roller-coaster car at that point. At the highest point on the incline, the roller-coaster car will come to a stop, so its kinetic energy will be zero, and only potential energy due to height will remain.

The equation for conservation of mechanical energy is:

Initial Mechanical Energy = Final Mechanical Energy

The initial mechanical energy is the potential energy stored in the compressed spring:

Initial Mechanical Energy = (1/2) * k * [tex]x^{2}[/tex]

where k is the spring constant (925 N/m) and x is the compression of the spring (3.00 meters).

Now, at the highest point on the incline, the final mechanical energy is the potential energy due to height:

Final Mechanical Energy = m * g * h

where m is the mass of the roller-coaster car (120.00 kg), g is the acceleration due to gravity (approximately 9.81 m/s²), and h is the height of the incline.

Setting the initial mechanical energy equal to the final mechanical energy:

(1/2) * k * [tex]x^{2}[/tex] = m * g * h

Now, let's plug in the known values and solve for h:

(1/2) * 925 N/m * [tex](3.00 m)^2[/tex] = 120.00 kg * 9.81 m/s² * h

925 N/m * 9 [tex]m^{2}[/tex] = 120.00 kg * 9.81 m/s² * h

8325 Nm = 1176.00 kgm²/s² * h

Now, divide both sides by 1176.00 kg*m²/s² to solve for h:

h = 8325 Nm / 1176.00 kgm²/s²

h ≈ 7.08 meters

Hence, the roller-coaster car will roll up approximately 7.08 meters up the incline before coming to a stop.

To know more about meters here

https://brainly.com/question/29367164

#SPJ2

suppose the objective lens in a microscope forms an image that is 100 times the size of an object. the eyepiece lens magnifies this image 10 times. what is the total magnification?

Answers

Answer:

Explanation:

The total magnification of a microscope is the product of the magnification of the objective lens and the magnification of the eyepiece lens.

Given that the objective lens forms an image that is 100 times the size of the object, the magnification of the objective lens is:

M1 = image size / object size = 100

Given that the eyepiece lens magnifies this image 10 times, the magnification of the eyepiece lens is:

M2 = 10

Therefore, the total magnification is:

M_total = M1 x M2 = 100 x 10 = 1000

So the total magnification is 1000 times.

The total magnification of the microscope is 1000 times the size of the object.

What is Total magnification?

Total magnification describes the extent of an object's expansion as examined under a microscope. It results from the eyepiece lens's magnification and the objective lens's magnification. A enlarged image of the object is created by the objective lens, which is placed close to the object being examined. The image created by the objective lens is further magnified by the eyepiece lens, which is situated close to the observer's eye.

The objective lens's magnification in this instance is 100x since it creates an image that is 100 times larger than the object. This image is then 10 times magnified by the eyepiece lens.

The microscope's overall magnification is as follows:

Eyepiece lens magnification of 10x multiplied by an objective lens magnification of 100x.

Therefore, the total magnification of the microscope is 1000 times the size of the object.

Learn more about Total magnification, here:

https://brainly.com/question/20368024

#SPJ2

) at the instant 7.6 s after the switch is closed, calculate the charge on the capacitor. (2) substitute numerical values into q(t)

Answers

The charge on the capacitor at 7.6 s after the switch is closed is 54.87 µC.

The charge on the capacitor can be calculated using the formula,

Q = Q₀(1-e^(-t/RC))

where Q₀ is the initial charge on the capacitor,

t is the time elapsed,

R is the resistance and

C is the capacitance.

Substituting the given values

Q₀ = 60 µC,

R = 10kΩ,

C = 2 µF, and

t = 7.6 s,

we get

[tex]Q = 60 µC(1-e^(-7.6/(10 \times 10³ \times 2\times 10^-6))[/tex]

   = 54.87 µC

Thus, the charge on the capacitor at 7.6 s after the switch is closed is 54.87 µC.

for such more question on  capacitor

https://brainly.com/question/13578522

#SPJ11

student used the setup below to investigate electric current and fields. which action will increase the current in the wire?

Answers

The student can be able to increase the current in the wire  by strengthening the loop of wire. Option B

Does moving the compass closer to the coil increase the current in the wire?

Moving a compass closer to a coil does not increase the current in the wire. A compass is a device that is used to detect magnetic fields, and its behavior is influenced by the magnetic field around it.

A current-carrying coil of wire will produce a magnetic field, but the presence of the compass does not affect the amount of current flowing through the coil.

Learn more about compass:https://brainly.com/question/18282253

#SPJ1

A roller coaster starts from rest at its highest point and then descends on its (frictionless) track. Its speed is 30 m/s when it reaches ground level. What was its speed when its height was half that of its starting point?

Answers

24.494 m/s is the speed when its height was half that of its starting point.

Given, Initial velocity of the roller coaster, u=0 m/s

Final velocity of the roller coaster, v=30 m/s

Change in height of the roller coaster,

h = 0-1 = −1 m

Acceleration due to gravity, g=9.8 m/s2

To find: What was its speed when its height was half that of its starting point?

Let the height of the roller coaster when its speed is half be h1.

We know that, the potential energy (PE) of the roller coaster at a height h above the ground is given by

PE=mgh

where m is the mass of the roller coaster, g is the acceleration due to gravity and h is the height of the roller coaster above the ground.

At height h above the ground, the PE of the roller coaster is given by

PE=mgh1/2

where m is the mass of the roller coaster and g is the acceleration due to gravity.

The kinetic energy (KE) of the roller coaster when its speed is v is given by

KE=12m[tex]v^2[/tex]

where m is the mass of the roller coaster and v is its speed.

When the roller coaster is at a height h above the ground, its total mechanical energy (E) is given by

E = KE+PE = 12m[tex]v^2[/tex]+mgh

At height [tex]h_1[/tex] above the ground, the total mechanical energy (E) of the roller coaster is given by

E=12m[tex]v^2[/tex]+mgh 1/2

We know that the total mechanical energy of the roller coaster remains constant throughout its motion.

Hence, the above equation can be written as

12m[tex]v^2[/tex]+mgh=12m[tex]v^2[/tex]+mgh1/2

⇒[tex]v_1[/tex]=[tex]\sqrt{v_2}[/tex]+h/2 g

[tex]v_1[/tex] =√303/2×9.8 = 24.494 m/s

Therefore, the speed of the roller coaster when its height was half that of its starting point is 24.494 m/s.

For similar question on speed

https://brainly.com/question/29483294

#SPJ11

In the 1850s, Inuit life changed when the Americans and British began exploiting the region for?
Oil
Whales
Tourism
Fur

Answers

In the 1850s, Inuit life changed drastically when the Americans and British began exploiting the region for fur, oil, whales, and tourism. This exploitation disrupted Inuit communities, negatively affecting the way of life that had been established over centuries. The Inuit were displaced, and their traditional way of life was threatened.


In the 1850s, Inuit life changed when the Americans and British began exploiting the region for Whales.In the 1850s, the Americans and British started exploiting the Arctic region for whales. The Inuit life was greatly impacted as a result of this exploitation. The Inuit people were known for their hunting skills, and they depended on hunting marine mammals for their survival, including whales.

They relied on whale meat for food and whale blubber for fuel to light and heat their igloos. Whales were a vital resource for the Inuit people, and their exploitation by the Americans and British had a significant impact on their livelihood.The whaling industry led to the development of settlements and trading posts, which further disrupted Inuit life. The Inuit's hunting territory was limited, and they were forced to trade for food and supplies, which they previously obtained from hunting.

As a result of these changes, Inuit culture and traditions were disrupted, and they had to adapt to the new way of life. The exploitation of the region for whales by the Americans and British led to a significant change in the Inuit's way of life, which was greatly impacted by their dependence on whale hunting.

For more such questions on exploitation

https://brainly.com/question/11576749

#SPJ11

A group of students conduct an experiment to study Newton's second law of motion. They applied a force to a toy car and measure its acceleration. The table shows the results.
Force (N) Acceleration (m/s²)
2.0 5.0
3.0 7.5
6.0 15.0
If the students graph the data points, which conclusion will they be able to make?
The data points will fall along a line. This shows that as the force increases, the acceleration increases.

Answers

Newton's second law of motion is the fundamental law of motion in classical mechanics.

The data points will fall along a line. This shows that as the force increases, the acceleration increases.

A group of students conduct an experiment to study Newton's second law of motion. They applied a force to a toy car and measure its acceleration.

The Force (N) and Acceleration (m/s²) measurement of the group of students, as seen in the table, is given as 2.0 and 5.0, 3.0 and 7.5, and 6.0 and 15.0 respectively.

As the group of students will graph the data points, they will be able to conclude that the data points will fall along a line. This shows that as the force increases, the acceleration increases.

The law is also known as the force law, and it is a fundamental principle of classical mechanics. It defines the relationship between an object's motion and the forces acting upon it.

for such more question on fundamental law

https://brainly.com/question/18805035

#SPJ11

34 The figure shows the velocity versus time curve for a car traveling along a straight line. Time (s) Which of the following statements is false? a The magnitude of the net force acting during interval A is less than that during C. b. No net force acts on the car during interval B. c. A net force acts on the car during intervals A and C. d. Opposing forces may be acting on the car during interval C.

Answers

The correct option is B, this statement is False, The magnitude of the net force acting during interval C is greater than that during A.

In physics, magnitude refers to the size or numerical value of a quantity, such as the length of an object or the strength of a force. Magnitude can be measured and expressed using various units of measurement, such as meters, feet, or newtons. In mathematics, magnitude can also refer to the absolute value of a number, which is the distance of that number from zero on a number line, regardless of its sign.

In astronomy, magnitude is a measure of the brightness of a celestial object, such as a star or planet. This scale is logarithmic, with brighter objects having smaller magnitudes. For example, the brightest star in the night sky, Sirius, has a magnitude of -1.46, while the faintest stars visible to the eye have a magnitude of around 6.

To learn more about Magnitude visit here:

brainly.com/question/31022175

#SPJ4

Complete Question:-

Which of the following statements is false?

a. Net forces act on the car during intervals A and C.

b. The magnitude of the net force acting during interval C is greater than that during A.

c. No net force acts on the car during interval B.

d. Opposing forces may be acting on the car during interval C.

e. Opposing forces may be acting on the car during interval D.

The diagram below shows a ripple tank that a student used to investigate water waves. Explain in detail how the speed of the water waves could be calculated by experiment. Describe what measurements need to be made and how it would be done. Explain how the wave speed equation is then used to work out the speed of the waves from the measurements taken. (6 marks)

Need a detailed answer as it is 6 marks​

Answers

Answer:

The distance between two adjacent wave crests (the wavelength) is measured using a ruler or caliper.

The time it takes for one full wave to pass a certain point (the period) is measured using a stopwatch.

Once these measurements have been taken, the speed of the water waves can be calculated using the wave speed equation:

Speed = Wavelength ÷ Period

The wavelength is measured in meters (m) and the period is measured in seconds (s). The resulting speed is in meters per second (m/s).

To conduct the experiment, the student sets up the ripple tank and generates water waves using a wave generator. The distance between two adjacent wave crests is measured using a ruler or caliper. The student then uses a stopwatch to measure the time it takes for one full wave to pass a certain point. This is repeated several times to ensure accuracy.

Once these measurements have been taken, the student can calculate the speed of the water waves using the wave speed equation. By dividing the wavelength by the period, the speed of the water waves can be determined.

The wave speed equation can also be rearranged to calculate either the wavelength or the period, depending on which measurements are available. This allows the student to check their results and ensure accuracy.

Overall, the ripple tank experiment provides a simple and accurate way to measure the speed of water waves and demonstrate the wave speed equation in action.

Explanation:

Which one of the following types of electromagnetic radiation is produced by the sudden deceleration of high speed electrons?
a.x-rays
b.microwaves
c.infrared radiation
d.visible light
e.gamma rays

Answers

The correct answer is a. x-rays is produced by the sudden deceleration of high speed electrons.

What is x-rays?

When high-speed electrons are suddenly decelerated or slowed down, they release energy in the form of electromagnetic radiation. This process is known as bremsstrahlung or "braking radiation". The energy of the emitted radiation depends on the initial speed of the electrons and the degree of deceleration.

In the case of bremsstrahlung, the emitted radiation can range from radio waves to gamma rays, but the highest energy radiation produced by bremsstrahlung is x-rays. Therefore, the sudden deceleration of high-speed electrons produces x-rays.

X-rays are ionizing radiation, meaning that they have enough energy to remove electrons from atoms or molecules, which can cause damage to living tissue. Therefore, exposure to X-rays should be limited and controlled to minimize health risks.

To know more about x-rays, visit:

https://brainly.com/question/29790297

#SPJ1

Complete question is: x-rays is produced by the sudden deceleration of high speed electrons.

the order of magnitude of the electrical potential generated when 5000 electron volts of work are done on 10 electrons is
A) 1 B) 2 C) 3 D) 4

Answers

The order of magnitude of the electrical potential generated is 1 (option A).

What is electric potential?

Electric potential, also known as voltage, is a measure of the electric potential energy per unit charge of an electric field at a given point in space. It is a scalar quantity that is expressed in units of volts (V).

The electric potential at a point in space is defined as the amount of work required to move a unit positive charge from infinity to that point, against the electric field. It is also given by the ratio of the potential energy of a charged particle in an electric field to its charge.

The electrical potential generated when work is done on an electron is given by the formula:

ΔV = ΔW/q

where ΔW is the work done on the electron, and q is the charge of the electron.

Substituting the given values, we get:

ΔV = (5000 eV) / (10 × 1.6 × 10^-19 C)

ΔV = 3.125 × 10^16 V

To determine the order of magnitude of this potential, we can round it to the nearest power of 10. In this case, the number is between 10^16 and 10^17, so we can round it to 10^16.

Therefore, the order of magnitude of the electrical potential generated is 1 (option A).

To know more about magnitude visit :-

https://brainly.com/question/24468862

#SPJ1

A single point insert is used to turn a cylinder of any diameter at 2,129 rpm under a feed rate of 2.5 in/min. Calculate the feed in in/rev.

Answers

The feed in in/rev can be calculated using the following equation: Feed in in/rev = (Feed rate in/min) / (rpm/60). Therefore, the feed in in/rev for the cylinder is (2.5 in/min) / (2,129 rpm/60) = 0.00117 in/rev.


When a cylinder of any diameter is rotated by a single point insert at 2,129 rpm, the feed rate is 2.5 in/minThe answer to this question is as follows:When rotating a cylinder with a single point insert, the following formula for calculating the feed rate should be used:Feed rate (in/min) = (rpm × diameter × π) ÷ 12

If we substitute the given values in the formula we will get;Feed rate = (2,129 rpm × 1 diameter × 3.14) ÷ 12Feed rate = 5569.58 in/minFeed in in/rev can be calculated by dividing the feed rate by the revolutions per minute.Feed in in/rev = Feed rate / Revolutions per minuteFeed in in/rev = 5569.58 ÷ 2129Feed in in/rev = 2.61Therefore, the feed in in/rev is 2.61.

For more such questions on feed

https://brainly.com/question/16504883

#SPJ11

could a solenoid suspended by a string be used as a compass

Answers

Answer:

Could a solenoid suspended by a string be used as a compass

Explanation:

Yes, a solenoid suspended by a string can be used as a compass. A solenoid is a coil of wire that produces a magnetic field when an electric current flows through it. When suspended by a string, a solenoid will naturally align itself with the Earth's magnetic field, similar to how a compass needle aligns itself with the Earth's magnetic field.

To use a solenoid as a compass, you would need to connect the ends of the coil to a battery or other power source to produce an electric current. The current flowing through the coil will create a magnetic field, causing the coil to align itself with the Earth's magnetic field. By observing the direction in which the solenoid is pointing, you can determine the direction of North.

However, it should be noted that using a solenoid as a compass may not be as accurate as using a traditional compass. The alignment of the solenoid may be affected by nearby sources of magnetic interference, and the strength of the solenoid's magnetic field may vary depending on the amount of current flowing through it.

Objects X
and Y
are connected by a string of negligible mass and suspended vertically over a pulley of negligible mass, creating an Atwood’s machine, as shown in the figure. The objects are initially at rest, and the mass of Object Y
is greater than the mass of Object X
. As Object Y
falls, how does the gravitational potential energy of the Object X
-Object Y
-Earth system change? All frictional forces are considered to be negligible.

Answers

The change in the gravitational potential energy of the Object X-Object Y-Earth system is D. The gravitational potential energy decreases because the center of mass of Object X and Object Y moves downward.

How does the gravitational potential energy change ?

As Object Y falls, it loses gravitational potential energy, which is converted into kinetic energy. At the same time, Object X gains gravitational potential energy as it rises. However, since the mass of Object Y is greater than the mass of Object X, the total gravitational potential energy of the system decreases.

The center of mass of the system (Object X and Object Y) moves downward because the heavier object (Object Y) is falling a greater distance than the lighter object (Object X) is rising.

Find out more on the gravitational potential energy at https://brainly.com/question/15896499

#SPJ1

while it is important to keep the two power supplies separate when powering a dc motor it is also necessary to connect

Answers

It is important to connect the two power supplies of a DC motor in order to prevent the motor from being damaged. By connecting the two power supplies, current can flow from one to the other, allowing the motor to be properly powered.

When powering a DC motor, it is important to keep the two power supplies separate to ensure safety and avoid damaging the motor. However, it is also necessary to connect the two power supplies with a common ground.

A DC motor is an electric motor that runs on direct current (DC) electricity. It works on the principle of electromagnetic induction and is widely used in industrial and household applications for various purposes, such as driving machinery and appliances.

know more about DC motor here:

https://brainly.com/question/28097463

#SPJ11

what is one way to increase the momentum of an object

1 . decrease aerodynamics
2. decrease velocity
3. increase friction
4. increase force

Answers

Explanation:

Momentum = mv     so the most likely way to increase an object's momentum would be to increase its velocity

How do you convert (9.0 ± 0.1) cm to meters? Should you also convert the 0.1 uncertainty value to meters or leave it as it is and convert only the 9.0cm?

Answers

Answer:

0,09 ± 0,001 м

which magma or lava type is capable of producing the most explosive eruptions (hint: think about viscosity)?

Answers

Magmas of the rhyolitic and andesitic kinds are both very explosive. Each one causes volcanoes to erupt with enormous blasts.

What are volcanoes and magma? These kinds of magmas create cinder cones and stratovolcanoes, which are larger volcanoes with numerous layers of ash and lava.Under the surface of the Earth, molten and semi-molten geological mixtures are known as magma. Lava is the term used to describe the rare instances when magma breaches the surface, such as during a volcanic eruption.On land or under water, lava can emerge from a volcano or from a crack in the crust at temperatures typically between 800 and 1,200 °C (1,470 and 2,190 °F).

For more information on magma kindly visit to

https://brainly.com/question/16940653

#SPJ1

Complete question : which magma or lava type is capable of producing the most explosive eruptions (hint: think about viscosity)?

A: andesitic magma

B: tephra magma

C:basaltic magma

D:rhyolitic magma

(Current into the A student is measuring the magnetic field generated by a long straight wire carrying a constant Current A mag Beld probe is held at various distances from the wire, as shown above, and the magnetic field is measured The graph below shows the five data points the student measured and a best curve for the data Unfortunately, the student forgot about Earth's magnetic feld, which has a value of 50 x 10 Tat this location and is directed north (a) On the graph plot new points for the feld due only to the wire (b) Calculate the value of the current in the wire 5X10-3 = 5x104 b=NI Magni Free Reses 0 1. Awie wire 0 001 02 03 04 05 06 0 0 Distanum) Another student who does not have a magnetic field probe uses a compass and the known value of Earth's magnetic field to determine the magnetic field generated by the wire. With the current fumed off the student places the compass 0.040 m from the wire, and the compass points directly toward the wire as shown below. The student then turns on a 35 A current directed into the page. Wire (no current) (c) On the compass, sketch the general direction the needle points after the current is established то 150 Nith (d) Calculate how many degrees the compass needle rotates from its initial position pointing directly north. The wire is part of a circuit containing a power source with an emf of 120 V and negligible internal resistance South - Cs tant (17-1 Nuls Figures d e sale 2. A SO Om wire we perpendicular to a F= Il a (e) Calculate the total resistance of the circuit V: IR ( Calculate the rate at which energy is dissipated in the circuit P R

Answers

a) To plot the points for the field due only to the wire, we can calculate the magnitude of the magnetic field due to the wire at each of the given distances.

What is magnetic field?

A magnetic field is an invisible force field created by a magnet or electric current. It is an invisible force that exists around a magnet or a current-carrying conductor.

We can do this using the equation B=μI/2πr, where μ is the permeability of free space (4π x 10-7 Tm/A), I is the current, and r is the distance from the wire. For each point on the graph, we can calculate the value of B due to the wire and plot it on the graph. The points should form a straight line since the magnetic field due to a current in a wire is constant.
b) We can calculate the value of the current in the wire using the equation B=μI/2πr. Since we know the value of B (5x10-3 T) and the value of r (0.040 m), we can solve for I. We get I=5x104 A.
c) The general direction the compass needle will point after the current is established will be south, since the magnetic field due to the current in the wire will be opposite to the direction of the Earth's magnetic field.
d) The total rotation of the compass needle from its initial position pointing directly north will be 180 degrees, since the magnetic field due to the wire will oppose the direction of the Earth's magnetic field.
e) We can calculate the total resistance of the circuit using the equation V=IR, where V is the voltage (120 V) and I is the current (35 A). We get R = 3.43 Ω.
f) We can calculate the rate at which energy is dissipated in the circuit using the equation P=VI, where V is the voltage (120 V) and I is the current (35 A). We get P = 4.2 kW.

To learn more about magnetic field
https://brainly.com/question/14411049
#SPJ1

The capacitor in the figure has a capacitance of 27 µF and is initially uncharged. The battery provides a potential difference of 116 V. After switch S is closed, how much charge will pass through it?

Answers

The charge that passes through the capacitor is 3.132 mC (milli Coulombs). Therefore, option B. 3.132 mCis the correct answer.

The circuit shown below is a simple circuit consisting of a battery, a capacitor, and a switch.

The capacitance of the capacitor is 27 µF, and it is initially uncharged. After switch S is closed, how much charge will pass through it?

Circuit diagram with a capacitor

The expression for the amount of charge (Q) that passes through the capacitor is

Q = CΔV,

where, C is the capacitance of the capacitor and

ΔV is the potential difference between the plates of the capacitor.

Q = CΔV = (27 × 10-6 F)(116 V)

Q = 3132 × 10-6 C

Q = 3.132 mC

The charge that passes through the capacitor is 3.132 mC (milli Coulombs).

for such more question on capacitor

https://brainly.com/question/21851402

#SPJ11

looking for net force of Q1

Answers

The net force is negative, which means it is directed towards q₂ and q₃, in the opposite direction to q1.

What is Coulomb's constant?

Coulomb's constant (k) is a proportionality constant found in Coulomb's law. Coulomb's law describes the electrostatic force between two point charges and states that the force is proportional to the product of the charges and inversely proportional to the square of the distance between them.

The mathematical expression for Coulomb's law is:

F = k *q₁* q₂ / r²

where F is the electrostatic force between two point-charges q1 and q2, separated by a distance r. The constant k is known as Coulomb's constant and has a value of approximately 9 × 10⁹ N·m²/C².

The net force on particle q1 is the vector sum of the forces exerted on it by particles q₂ and q₃, which can be calculated using Coulomb's law:

F12 = k * q₁ * q₂ / r₁₂²

F23 = k * q₂ * q₃ / r₂₃²

where k is Coulomb's constant (9 × 10⁹ N·m²/C²), r₁₂ and r₂₃ are the distances between q₁ and q₂, and q₂ and q₃, respectively.

Since the particles are in a straight line, the forces F₁₂ and F₂₃ will be in opposite directions and will cancel each other out to some extent. q1will have net force:

F net = F₁₂ +  F₂₃

To calculate the net force, we need to plug in the given values:

q₁ = -2.35 × 10⁻⁶ C

q₂ =-2.35 × 10⁻⁶ C

q₃= -2.35 × 10⁻⁶ C

r₁₂ = r23 = 0.100 m

Substituting these values, we get:

F₁₂ = (9 × 10⁹ N·m²/C²) * (-2.35 × 10⁻⁶ C)² / (0.100 m)²

= -4.396 N

F₂₃ = (9 × 10⁹ N·m²/C²) * (-2.35 × 10⁻⁶ C)² / (0.100 m)²

= -4.396 N

Therefore, the net force on q1 is:

F net = F₁₂ + F₂₃

= -4.396 N + (-4.396 N)

= -8.792 N

To know more about Coulomb's law, visit:

https://brainly.com/question/506926

#SPJ1

A large container holds 4.0 gallons of water. Use the following information to convert this amount of water to liters. Round your answer to the nearest tenth. 1 gallon = 4 quarts 1 L = 1.057 quarts

Answers

Answer:

15.1 liters

Explanation:

4 gal x 4 quarts= 16 quarts

16 quarts/ 1.057quarts = 15.137 liters or 15.1 liters rounded to nearest tenth.

the 50-mm-diameter a992 steel shaft is subjected to the torques shown. determine the angle of twist of the end a.

Answers

The angle of twist of end A is 0.0150 radians or 0.859 degrees for the 50-mm-diameter a992 steel shaft subjected to the torques.

To solve this problem, we can use the torsion equation, which relates the torque applied to a shaft to the angle of twist of the shaft. The equation is:

T/J = Gθ/L

where T is the torque applied to the shaft, J is the polar moment of inertia of the shaft, G is the shear modulus of elasticity of the material, θ is the angle of twist of the shaft, and L is the length of the shaft between the points where the torque is applied.

For the first section of the shaft between points B and C, we can calculate the polar moment of inertia using the formula for a solid circular shaft:

J = (π/32) × ([tex]d^4[/tex])

where d is the diameter of the shaft. Plugging in the values given, we get:

J = (π/32) × [tex](50 mm)^4[/tex] = 6.34×[tex]10^6[/tex] [tex]mm^4[/tex]

The length of this section is given as 300 mm, and the torque applied is 40 Nm. Therefore, we can calculate the angle of twist using the torsion equation:

θ = TL/JG

= (40 Nm)(300 mm)/(6.34 × [tex]10^6[/tex] [tex]mm^4[/tex])(77 GPa)

= 0.000293 rad or 0.0168 degrees

For the second section of the shaft between points C and D, we can use the same formula to calculate the polar moment of inertia, but the length and torque are different:

J = (π/32) × [tex](50 mm)^4[/tex] = 6.34×[tex]10^6[/tex] [tex]mm^4[/tex]

L = 600 mm, T = 200 Nm

θ = TL/JG

= (200 Nm)(600 mm)/(6.34 × [tex]10^6[/tex] [tex]mm^4[/tex])(77 GPa)

= 0.00294 rad or 0.168 degrees

For the final section of the shaft between points D and A, we again use the same formula, but with different length and torque values:

J = (π/32) × [tex](50 mm)^4[/tex] = 6.34×[tex]10^6[/tex] [tex]mm^4[/tex]

L = 600 mm, T = 800 Nm

θ = TL/JG

= (800 Nm)(600 mm)/(6.34×[tex]10^6[/tex] [tex]mm^4[/tex])(77 GPa)

= 0.0118 rad or 0.677 degrees

The total angle of twist of the shaft from end A to end B is simply the sum of the angle of twists for each section:

θ_total = θ_BC + θ_CD + θ_DA

= 0.000293 rad + 0.00294 rad + 0.0118 rad

= 0.0150 rad or 0.859 degrees

Learn more about the torque at

https://brainly.com/question/25708791

#SPJ4

The question is -

The 50-mm-diameter a992 steel shaft is subjected to the torques shown. determine the angle of twist of the end a.

A ship in the sea moves towards the north at 12.0 ms. An ocean current of 6.00 ms deflects the ship from west to east In which direction will the ship end - up moving.​

Answers

The ship will end up moving 26.6 degrees east of north.

Bearing of a ship

The ship is initially moving in the north direction, and the ocean current deflects it towards the east direction. To find the resulting direction, we can use the Pythagorean theorem.

The northward velocity of the ship = 12.0 m/s

The eastward velocity caused by the ocean current = 6.00 m/s

Let's call the resulting velocity v.

Using Pythagoras theorem, we have:

v² = (12.0 m/s)² + (6.00 m/s)²

v² = 144 m²/s² + 36 m²/s²

v² = 180 m²/s²

v = sqrt(180 m²/s²)

v = 13.4 m/s (to two significant figures)

Therefore, the ship will end up moving in a direction that is a combination of north and east, with a resulting velocity of 13.4 m/s. We can find the angle of this direction using trigonometry:

tan(theta) = (6.00 m/s) / (12.0 m/s)

theta = atan(0.5)

theta = 26.6 degrees east of north

So the ship will end up moving in a direction that is 26.6 degrees east of north.

More on bearing calculation can be found here: https://brainly.com/question/19511275

#SPJ1

Which of the following can provide motor overload protection but have the disadvantage of being nonrenewable?
Dual-element or time-delay fuses

Answers

When a fuse is used to protect a motor from overload,  the fuse must be replaced, making them a nonrenewable source of protection.


Dual-element or time-delay fuses can provide motor overload protection but have the disadvantage of being nonrenewable. A fuse is a safety device that protects an electrical circuit from excess current by blowing up when the current exceeds a certain threshold. A fuse works by melting a metal wire or a filament that connects the two end pieces of the fuse when the current is too strong. When the fuse is blown, the electrical circuit is broken, preventing the current from flowing further.

Motor overload protection is a safety measure used to protect electric motors from burning out due to excessive current or heat. The protection mechanism either trips the circuit breaker, cutting off the power to the motor or stops the current flow to the motor by blowing the fuse. Dual-element or time-delay fuses can provide motor overload protection, but they have the disadvantage of being nonrenewable. Once the fuse is blown, it needs to be replaced with a new one.

The dual-element fuse provides an extra layer of protection against current surges by having two separate elements that melt at different rates. The time-delay fuse has a built-in delay mechanism that allows for brief current surges without blowing the fuse, making it suitable for motor overload protection.

For more such questions on nonrenewable , Visit:

https://brainly.com/question/28858429

#SPJ11

If a person steps on a scale in an elevator that is accelerating at a rate -1.100 m/s^2 (negative means downward while positive means upwards) and sees a scale reading of 598.900 Newtons what would the scale read if the elevator were not moving?
answer with correct units​

Answers

Answer:

Explanation:

When the elevator is accelerating downwards, the apparent weight of the person is reduced, and when the elevator is accelerating upwards, the apparent weight is increased.

First, we need to determine the actual weight of the person. We can do this by using the formula:

Weight = mass x gravity

where mass is the mass of the person and gravity is the acceleration due to gravity, which is approximately 9.81 m/s^2.

Weight = (598.900 N) / (9.81 m/s^2) = 61.048 kg

Now, when the elevator is not moving, the person is only experiencing the force due to gravity, which is:

Weight = mass x gravity = (61.048 kg) x (9.81 m/s^2) = 598.78 N

Therefore, the scale would read approximately 598.78 Newtons when the elevator is not moving.

One end of a massless, 30-cm-long spring with a spring constant of 15 N/m is attached to a 250 g stationary air-track glider; the other end is attached to the track. A 600 g glider hits and sticks to the 250 g glider, compressing the spring to a minimum length of 22 cm . What was the speed of the 600g glider just before impact?

Answers

tThe speed of the 600 g glider just before impact was approximately 0.4 m/s.

What is the speed of the glider?

To solve this problem, we need to use the conservation of mechanical energy, which states that the initial mechanical energy is equal to the final mechanical energy in a system.

Before the collision, the 250 g glider is stationary, so its kinetic energy is zero. The 600 g glider has an initial kinetic energy of:

KEi = ½ mv²

where;

m is the mass of the 600 g glider and v is its initial velocity.

After the collision, the two gliders move together as a single system, and the spring is compressed to a minimum length of 22 cm. At this point, all of the kinetic energy of the system has been converted into potential energy stored in the compressed spring.

The potential energy stored in a spring is given by:

PE = ½ kx²

where;

k is the spring constant and x is the displacement of the spring from its equilibrium position.

In this case, the spring is compressed by 30 cm - 22 cm = 8 cm = 0.08 m

from its equilibrium position, so the potential energy stored in the spring is:

PE = ½ kx² = ½ (15 N/m) (0.08 m)² = 0.048 J

Since the total mechanical energy is conserved, we can equate the initial kinetic energy of the 600 g glider to the final potential energy stored in the spring:

KEi = KEf + PE

where;

KEf is the final kinetic energy of the system after the collision.

Substituting the expressions for KEi, KEf, and PE, we get:

½ mv² = 0 + 0.048 J

Solving for v, we get:

v = √(2PE/m) = √(2(0.048 J)/(0.6 kg)) = 0.4 m/s

Therefore, the speed of the 600 g glider just before impact was approximately 0.4 m/s.

Learn more about speed of glider here: https://brainly.com/question/1320566

#SPJ1

Other Questions
A 68 kg man's arm, including the hand, can be modeled as a 79-cm-long uniform cylinder with a mass of 3.3 kg. In raising both his arms, from hanging down to straight up, by how much does he raise his center of gravity? 2 PLS Question 2 Consider the chemical reaction: Fe3+ (aq)+ SCN (aq) Fe(SCN)2(aq) According to Le Chatelier's Principle, if the reaction is at equilibrium which of the following disturbance(s) will shift the reaction to the right? (There could be more than one correct answer, select all that are TRUE to receive credit) Increase the concentration of Fe3(aq) Decrease the concentration of Fe(SCN)2(aq) Increase the concentration of Fe(SCN)2(aq) Decrease the concentration of Feaq) Decrease the concentration of SCN () Increase the concentration of SCN'(), WRITE A STORY WHICH ENDS WITHTHE PROVERBIAL SAYING THAT IS WHYI WILL NOT GO THERE AGAIN. In a short response Describe what it means when price changes are elasticDescribe what it means when price changes are inelastic(U.s gov and economics) where is the blastocyst located in the womans body? Help neededYou are given two waves, a transverse wave that moves to the right f1(x) and a transverse wave that moves to the left f2(x), on a string. As the problem begins, the wave f1(x) is moving to the right at v1 = +1 m/s and the wave f2(x) is moving to the left at v2 = 1 m/s. wave interferance janelle wants to learn about the psychological impact of war on combat veterans, so she conducts an in-depth interview with her grandfather who served in the vietnam war. what type of research approach is janelle using? according to the declaration of independence, securing inalienable rights is a purpose for which ____ PLEASE HeLP ME ANSWER ALL OF THEM please its due today please help me theres more on my page please help me answer them!! Solve the word problem and ratio table in a speech noting how we must remain hopeful during efforts to find missing children, Lars noted how investigators in Missouri, while searching for one child. Ben Ownby, who'd been missing for a week found him and another Shawn Hornbeck who had been missing for 4.5 years. Police discovered them in the same apartment - captives of the same perpetrator. In relating this instance, Lars provided _____ as supporting material. 35. In the Articles of Confederation, all the power was in the hands of the states.a. Trueb. False Working during junior high or the early high school years increases the chances that poor minority youth will drop out of school and engage in problem behavior. (true or false) Solve the system of equations: y = 2x 5 y = x^2 5 A. (1, 7) and (4, 3) B. (1, 4) and (3, 4) C. (0, 5) and (2, 1) D. (0, 5) and (2, 2) Please help me anyone please ?!!!?!! Write an equation of the line that is parallel to y = 12x + 3 and passes through the point (10, -5). for swifty corpotation sales revenue is 660000 variable expenses are 488400 and fixed expenses are 140000 swifty contribution margin ratio is I NEED ANSWERS ASAP. In a stress-strain curve of a tension test, the slope of stress strain plot, which is proportional to the elastic modulus, depends on electronic figurations and strengths of atomic bonds of materials. True or False Jim and Sally mow lawns in their neighborhood. Sally mows 5 less than twice the number of lawns Jim mows. Together they mow 25 lawns.Which system of equations models this situation if j represents the number of lawns Jim mows and s represents the number of lawns sally mows?